A new device uses the global positioning system to determine a cow's location and, when a cow strays outside of its p...

cjahangiri on September 27, 2019

Why is the answer not A?

Can you explain how you got B to be the correct answer.

Reply
Create a free account to read and take part in forum discussions.

Already have an account? log in

Irina on September 27, 2019

@cjahangiri,

The maker predicts that the ranchers will purchase device at its current price, whereas (A) suggests that the price will go down significantly. (A) thus contradicts the conclusion of the argument. (B) tells us that cattle follow the lead of the same few members of the herd, meaning ranchers only need to purchase few of the devices to keep cattle at their pasture, making it overall a reasonable investment even at the current price. Notice that the argument only tells us that outfitting ALL of the cattle is far more expensive than other means, such as building a fence, further suggesting that the cost of outfitting only a few cows is likely on par or lower than other means.

Let me know if this helps and if you have any further questions.